Trapezoidal Approximation Help

  • Thread starter Thread starter opus
  • Start date Start date
  • Tags Tags
    Approximation
Click For Summary
The discussion focuses on using the trapezoidal rule to approximate the integral of 1/x from 1 to 2 with n=4 subintervals. The user calculates the interval length as Δx=0.25 and sets up the trapezoidal approximation formula. However, they mistakenly evaluate the function at incorrect points (1/4, 1/2, and 3/4) instead of the correct points (1, 1.25, 1.5, and 1.75). Upon realizing the error, they acknowledge the need for correction and express gratitude for the assistance. The correct integral approximation should yield a value closer to 0.697 instead of the erroneous 2.02.
opus
Gold Member
Messages
717
Reaction score
131

Homework Statement


Approximate each integral using the trapezoidal rule using the given number for ##n##.
##\int_1^2 \frac{1}{x}dx## where ##n=4##

Homework Equations


Trapezoidal Approximation "Rule":

Let ##[a,b]## be divided into ##n## subintervals, each of length ##Δx##, with endpoints at ##P={x_0,x_1,x_2,...x_n}##
Set ##T_n=\frac{1}{2}Δx\left[f(x_0)+2f(x_1)+2f(x_2)+...+2f(x_{n-1})+f(x_n)\right]##
Then,
##\lim_{n \rightarrow +\infty}T_n = \int_a^b f(x)dx##

The Attempt at a Solution


(i) ##n=4## and my intervals lengths are ##Δx=\frac{b-a}{n}=\frac{1}{4}##

(ii) ##\int_1^2 \frac{1}{x}dx ≈ \frac{1}{2}⋅\frac{1}{4}\left[f(1)+2f(1/4)+2f(1/2)+2f(3/4)+f(2)\right]##

##f(1)=1##
##2f(1/4)=8##
##2f(1/2)=4##
##2f(3/4)=\frac{8}{3}##
##f(2)=\frac{1}{2}##

Plugging the values into ##T_n##, I get ##\int_1^2 \frac{1}{x}dx ≈ 2.02##
The correct solution is 0.697, and I can't for the life of me see where I went wrong.

Could I get an extra pair of eyes on this?
 
Physics news on Phys.org
Why are you evaluating at 1/4, 1/2, and 3/4? They are not in your interval.
 
  • Like
Likes opus
I am a fool. Should be 1 1/4 not 1/4 etc. Jeez. Thank you!
 
Question: A clock's minute hand has length 4 and its hour hand has length 3. What is the distance between the tips at the moment when it is increasing most rapidly?(Putnam Exam Question) Answer: Making assumption that both the hands moves at constant angular velocities, the answer is ## \sqrt{7} .## But don't you think this assumption is somewhat doubtful and wrong?

Similar threads

  • · Replies 4 ·
Replies
4
Views
1K
  • · Replies 1 ·
Replies
1
Views
1K
  • · Replies 2 ·
Replies
2
Views
2K
  • · Replies 9 ·
Replies
9
Views
2K
  • · Replies 10 ·
Replies
10
Views
2K
  • · Replies 4 ·
Replies
4
Views
3K
  • · Replies 13 ·
Replies
13
Views
1K
Replies
17
Views
2K
  • · Replies 1 ·
Replies
1
Views
2K
Replies
5
Views
2K